Fourier Series Help: Find Steady State Solution of Diff Eq

Click For Summary
The discussion focuses on finding the steady state periodic solution of the differential equation x'' + 10x = F(t), where F(t) is defined piecewise with a period of 4. The user is attempting to derive the general Fourier series for F(t) and has correctly identified that the sine terms can be eliminated due to the even nature of F(t). They have calculated the coefficient a(0) as 0 and are working on the coefficients a(n), arriving at a(n) = [6/npi]*[sin(npi/2)], noting that it alternates in sign for odd n and is zero for even n. The user seeks clarification on whether the a(n) coefficients can only be positive and expresses uncertainty about the behavior of the series coefficients.
Giuseppe
Messages
42
Reaction score
0
Can anyone help me out with this?

Find the steady state periodic solution of the following differential equation.

x''+10x= F(t), where F(t) is the even function of period 4 such that
F(t)=3 if 0<t<1 , F(t)=-3 if 1<t<2.


Im basically just having a problem findind the general Fourier series for F(t).
I know how to do the latter part of the problem.

My work so far: Knowing this is even, I can eliminate the sin part of the Fourier series. So in general I need to solve for the series cofficients of a(0) and a(n)

for a(o) I get 0. Which makes sense too, even just by inspection of the graph of the function.

My problem is with a(n). My final result is [6/npi]*[sin(npi/2)]. How do I express that second term in my answer. I noticed that the sign alternates every other odd number. a(n) =0 for every even number.

Thanks a bunch
 
Physics news on Phys.org
My new basis of thought is that the a(n) Fourier coefficient can only be positive 6/npi... is this correct... if figure when you add the negative term, the coefficient becomes zero again. At no point can it be negative.

Correct me if I am wrong.

Thanks
 
Question: A clock's minute hand has length 4 and its hour hand has length 3. What is the distance between the tips at the moment when it is increasing most rapidly?(Putnam Exam Question) Answer: Making assumption that both the hands moves at constant angular velocities, the answer is ## \sqrt{7} .## But don't you think this assumption is somewhat doubtful and wrong?

Similar threads

  • · Replies 3 ·
Replies
3
Views
2K
Replies
3
Views
2K
  • · Replies 1 ·
Replies
1
Views
2K
  • · Replies 1 ·
Replies
1
Views
2K
  • · Replies 3 ·
Replies
3
Views
2K
  • · Replies 1 ·
Replies
1
Views
2K
  • · Replies 6 ·
Replies
6
Views
2K
  • · Replies 2 ·
Replies
2
Views
2K
  • · Replies 6 ·
Replies
6
Views
2K
  • · Replies 16 ·
Replies
16
Views
2K